Bài tập luyện thi học sinh giỏi Toán Lớp 11 - Nguyễn Hồng Hạnh (Có đáp án)

docx 20 trang nhungbui22 11/08/2022 2070
Bạn đang xem tài liệu "Bài tập luyện thi học sinh giỏi Toán Lớp 11 - Nguyễn Hồng Hạnh (Có đáp án)", để tải tài liệu gốc về máy bạn click vào nút DOWNLOAD ở trên

Tài liệu đính kèm:

  • docxbai_tap_luyen_thi_hoc_sinh_gioi_toan_lop_11_nguyen_hong_hanh.docx

Nội dung text: Bài tập luyện thi học sinh giỏi Toán Lớp 11 - Nguyễn Hồng Hạnh (Có đáp án)

  1. Câu 1. Ký hiệu x là số nguyên lớn nhất không vượt quá x. Giải phương trình x2 1 x x 2015 0. Hướng dẫn giải Ta có x 0. x2 x 2015 x2 x 2015 pt x x 1 x x 2015. x x x a ¢ a 2015 x2 a 1 x 2015 0 a 1 a 1 2 8060 x * 2 Do a 2015 x2 a 1 x 2015 0 a 1 a 1 2 8060 x 2015 (t/ m); 2 a 1 a 1 2 8060 a 1 a 1 2 4a 2015 loai 2 2 2  a 1 a 1 8060 Vậy S ; a ¢ ;a 2015 2  Câu 2. Tìm tất cả các đa thức P x ¡ x thỏa mãn điều kiện P P x x P x P x 1 ,x ¡ . Hướng dẫn giải Giả sử deg P x n . So sánh bậc của x trong hai vế của (1) ta được n2 2n n 0 . n 2 2 c 0 Khi n 0 ta được đa thức hằng P x c . Thay vào (1) ta được c c . c 1 Vậy P x 0 và P x 1 là các đa thức hằng thỏa mãn yêu cầu. Khi n 2 ta giả sử P x ax2 bx c với a 0 . So sánh hệ số cao nhất trong (1) ta được a3 a2 . Do a 0 nên ta có a 1 . Vậy ta có P x x2 bx c . Thế vào (1) kiểm tra thấy thỏa mãn. Kết luận: P x 0 , P x 1 và P x x2 bx c . Câu 3. Tìm tất cả các số nguyên dương n sao cho (n - 1)! không chia hết cho n2. Hướng dẫn giải Nhận xét rằng khi n là số nguyên tố thì do (n - 1) < n nên (n - 1)! hiển nhiên không chia hết cho n, và do đó không chia hết cho n2. Ta sẽ tìm n không nguyên tố thỏa (n - 1)! không chia hết cho n2. Ta có: (n 1)!  n2 n! n3 . Điều này xảy ra khi và chỉ khi tồn tại ít nhất một ước số p của n sao cho bậc của p (số mũ lũy thừa của p trong phân tích thừa số nguyên tố) trong n! là bé hơn bậc của p trong n3
  2. Giả sử n pt.k (với (p, k)=1). Theo lí luận trên ta có bất đẳng thức: n n n 3t (*) 2 3 p p p n n n n Suy ra: 3t 3t k.( pt 1 pt 2 1) p 2 3 t p p p k( pt 1) 1.(2t 1) 3t ( ). Suy ra: 3t 2t 1 t {1,2,3} p 1 2 1 Ta xét 3 trường hợp và dùng các phép thử lại để làm rõ kết quả bài toán • TH1: t = 1. Ta có: ( ) 3 k . Suy ra k 2 hoặc k 3 (Do k 1 thì n trở thành số nguyên tố) + Với k = 2: n 2 p (p nguyên tố). 2 p 2 p Thử lại: p = 2 thì n = 4 (thỏa); : (*) 3 (đúng) p 2 2 2 3 p p + Với k = 3: n 3p (p nguyên tố) Thử lại: p = 2 thì n = 6 (thỏa); p = 3 thì n = 9 (thỏa); p 5 : 3p 3p (*) 3 3 3 (sai) 2 p p • TH2: t = 2. Ta có ( ) 6 k( p 1) . Suy ra k 1 hoặc k 2 (Do ( p 1) 3 ) + Với k = 1, ta được ( p 1) 6 p {2,3,5} n {4,9,25} . Thử lại ta chọn: n = 4, n = 9. + Với k = 2, ta được ( p 1) 3 p 2 n 8 . Thử lại ta thấy n = 8 thỏa mãn. 2 • TH3: t = 3. Ta có ( ) 9 k( p p 1) . Suy ra k 1 (Do p2 p 1) 7 ) + Với k = 1, ta được ( p2 p 1) 9 p 2 n 8 (thỏa) Vậy tập tất cả các giá trị của số tự nhiên n thỏa (n 1)! n2 là {p , 2 p , 8 , 9 } với p nguyên tố. Câu 4. Cho 2k 1 số nguyên lẻ a0 ,a1, L,a2k (k ¥ ) . Chứng minh rằng phương trình 2k 2k 1 a2k x a2k 1x L a1x a0 0 không có nghiệm hữu tỷ. Hướng dẫn giải p Giả sử phương trình a x2k a x2k 1 L a x a 0 (1) có nghiệm hữu tỷ x , 2k 2k 1 1 0 q ( p,q) 1. p Thay x vào (1) ta có a p2k a p2k 1q L a pq2k 1 a q2k 0 (2) q 2k 2k 1 1 0 2k Suy ra a2k p  q , vì ( p,q) 1 nên a2k  q . Tương tự ta có a0  p . Vì a2k , a0 là những số lẻ nên p,q lẻ. Vế trái của (1) là tổng của 2k 1 số lẻ vì vậy đẳng thức (1) không xảy ra nên phương trình không có nghiệm hữu tỷ.
  3. Câu 5. Cho tập S gồm tất cả các số nguyên trên trong đoạn [1;2014]. Gọi T là tập hợp gồm tất cả các tập con không rỗng của S. Với mỗi tập hợp X T , ký hiệu m(X ) là trung bình cộng của tất cả m(X ) các số thuộc X . Đặt m  (ở đây tổng được lấy theo tất cả các tập hợp X T ). Hãy |T | tính giá trị của m. Hướng dẫn giải Với mỗi x [1,2, , 2014], đặt mk m(X) ở đây tổng được lấy theo tất cả các tập hợp X T mà | X | k . k 1 Xét số a bất kỳ thuộc S, suy ra a có mặt trong C2013 tập X T mà | X | k . k 1 k 1 Suy ra kmk (1 2 2014)C2013 1007.2015.C2013 2014 2014 k 1 2014 2014 C2013 2015 k 2015 k Do đó m(X)  mk 1007.2015  C2014  C2014 k 1 k 1 k 2 k 1 2 k 1 2015 (22014 1) 2 2015 Mà |T | (22015 1) m 2 Cách 2. Xây dựng song ánh từ T vào T như sau X T f (X ) {2015- x / x X} m(X ) m( f (X )) 2015 Suy ra 2m(X) m(X) m(f(X)) | T |.2015 m(X) 2015 Suy ra m  | T | 2 Câu 6. Tìm tất cả các số nguyên dương k để phương trình x2 y2 kxy x y 0 có nghiệm nguyên dương. Giải phương trình nghiệm nguyên dương với k nhỏ nhất tìm được. Hướng dẫn giải Tìm k (dùng kĩ thuật pt Markov). +) Gọi k là một giá trị thỏa mãn. Với k đó, gọi (a,b) là nghiệm nguyên dương của pt đã cho sao cho a b nhỏ nhất. KMTTQ coi a b . Khi đó ta có đẳng thức a2 (kb 1)a b2 b 0 . 2 2 +) Xét pt bậc 2: x (kb 1)x b b 0 , dễ thấy pt có 1 nghiệm x1 a , nên có nghiệm 2 x2 a ' , với a a ' kb 1(1) và a.a ' b b (2). Do a,b,k đều nguyên dương nên 2 đẳng thức trên chứng tỏ a ' cũng nguyên dương. Điều đó suy ra (a ',b) cũng là một nghiệm nguyên dương của pt đã cho, do tính “nhỏ nhất” của (a,b) nên a ' a . Kết hợp (2) được a2 a.a ' b2 b. Lại có a b nên b2 a2 b2 b (b 1)2 , suy ra a2 b2 hay a b . 2 +) Kết hợp (1)(2) được k 2 là nguyên dương, từ đây a = 1 hoặc 2, tương ứng k chỉ có a thể là 3 hoặc 4. +)(1đ) Giải pt
  4. Với k=3 là giá trị nhỏ nhất tìm được, pt cần giải tương đương x2 (3y 1)x y2 y 0 Để pt có nghiệm nguyên thì biệt thức là số chính phương hay 5y2 10y 1 t2 t2 5( y 1)2 4 . Bẳng cách giải pt kiểu Pell ta được nghiệm (t,y) thỏa mãn t ( y 1) 5 (u v 5)(9 4 5)n tương ứng với 3 nghiệm riêng (u,v) =(1,1), (4,2), (11,5) trong đó n N * (để thỏa mãn y nguyên dương) Từ đó ta có công thúc cụ thể cho giá trị y của nghiệm (x,y) (tất nhiên nguyên dương), còn giá trị x (3y 1 t) / 2 (đảm bảo là số nguyên dương). n n 1 Câu 7. Cho đa thức P x a0 x a1x an với hệ số thực. Chứng minh rằng nếu tất cả các n i 1 i 1 nghiệm của P x là số thực và phân biệt thì a2 . a a ,i 1,2, ,n 1. i n i i i 1 i 1 Hướng dẫn giải Xét đa thức Q y yn P y 1 có tất cả các nghiệm là số thực và phân biệt. Hơn nữa phương n 2 2 trình bậc 2 Q y n 2 n 3 4.3 n n 1 an y 2 n 1 an 1 y 2an 2 cũng có các 2 2 nghiệm thực phân biệt, suy ra n 1 an 1 2n n 1 anan 2 0 Với i n 1 thì (1) đúng, cũng có nghĩa mọi đa thức thỏa mãn đề bài thì hệ số của x thỏa mãn (1). n i 1 i 1 i 2 Với mỗi i 1,2, ,n 2 , xét đa thức P x b0 x b1x bi 1x bi x bi 1 cũng có tất cả các nghiệm là số thực và phân biệt. 2 i 1 Áp dụng BĐT cho b ta có b2 b b . i i i i 1 i 1 Trong đó bi 1 n i 1 4.3ai 1,bi n i 3.2ai , bi 1 n i 1 2.1ai 1 2 2 i 1 Suy ra n i 3.2ai n i 1 2.ai 1 n i 1 4.3ai 1 i i 1 n i 1 Hay a2 a a i i n i i 1 i 1 Ta có điều phải chứng minh. * 3 * Câu 8. Cho dãy số an : a1 ¥ ,an 1 an 2019,n ¥ . Chứng minh có nhiều nhất 1 số hạng của dãy là số chính phương. Hướng dẫn giải So sánh đồng dư của an , an 1 và an 2 theo modun 4 ta có (chú ý 2019  3 mod 4 ) Một số chính phương khi chia 4 có số dư là 0 hoặc 1. Vì vậy từ số hạng thứ 3 trở đi, dãy không có số chính phương nào.
  5. 2 2 Nếu cả a1 và a2 đều chính phương, giả sử a1 a ,a2 b , suy ra b2 a6 2019 b a3 b a3 2019 hơn nữa khi phân tích 2019 thành tích chỉ có 2 cách 2019 1.2019 3.673 . b a3 1 b 1010 Trường hợp 1: 3 3 , vô lí do 1009 không là lập phương. b a 2019 a 1009 b a3 3 b 338 Trường hợp 2: 3 3 , vô lí do 335 không là lập phương. b a 673 a 335 Vậy điều giả sử sai, nghĩa là dãy trên có nhiều nhất 1 số chính phương. Câu 9. Với n là số nguyên dương, một tập con của tập 1,2,3, ,n được gọi là tốt nếu sau khi ta sắp xếp thứ tự tăng các phần tử của nó thì thu được các số lẻ, chẵn, lẻ, theo thứ tự. Ví dụ các tập con tốt là 1,4,5,6, 3,4,7, tập  . Tập 2,3,4,7không là tập con tốt do nó bắt đầu bởi số chẵn. Tính số tập con tốt của tập 1,2,3, ,n . Hướng dẫn giải Gọi fn là số tập con tốt của 1,2,3, ,n . Ta lập hệ thức truy hồi của fn . + Nếu tập con tốt của 1,2,3, ,n không lấy n thì fn fn 1 . + Nếu tập con tốt của 1,2,3, ,n lấy n thì fn fn 2 . Vậy ta có fn fn 1 fn 2 . Hơn nữa f1 2, f2 3 1 5 Phương trình đặc trưng x2 x 1 0 x 2 n n 1 5 1 5 Suy ra fn A B 2 2 1 5 1 5 2 2 5 1 A B 2 2 2 A Thay 2 giá trị đầu ta được 2 2 5 5 1 5 1 5 2 A B 3 B 2 2 5 5 Suy ra n n n 1 n 1 2 2 5 1 1 5 2 1 5 2 5 1 1 5 1 1 5 fn 5 5 2 5 5 2 5 2 5 2 Câu 10. Cho đa thức P x x3 14x2 2x 1. Chứng minh rằng với mỗi x Z tồn tại số tự nhiên n sao cho ta luôn có P P P x x101.   n Hướng dẫn giải Cho x, y là hai số nguyên bất kỳ, ta chứng minh x  y mod101 P x  P y mod101 Thật vậy, hiển nhiên x  y mod101 P x  P y mod101 .
  6. 2 2 Biến đổi 4 P x P y 4 x y x xy y 14x 14 y 2 x y 2x y 14 2 3 y 29 2 101 2 y 27 , x  y mod101 1 Do đó, nếu P x  P y mod101 2 2 2x y 14 3 y 29  0 mod101 2 Nếu xảy ra (1) ta có ngay điều phải chứng minh. Nếu xảy ra (2), thì có hai khả năng 2 2 y 29 101 y 29;101 1, mà 2x y 14  3 . y 29 suy ra 3 là số chính phương mod 101 nên sử dụng ký hiệu Legendre có 3 3 101 2 1 1 (mâu thuẫn) 101 101 3 3 y 29101, thu được 2x y 14101, từ đó suy ra x  y  29 mod101 . Như vậy trong mọi trường hợp ta đều có x  y mod101 P x  P y mod101 . Xét 102 số P x , P P x , , P P P x . Theo nguyên lý Drichlet, tồn tại   102 m,n 1;2; ;102, m n sao cho P P P x  P P P x mod101 .     m n Từ nhận xét trên rút được P P P x  x mod101 .   m n Vậy suy ra điều phải chứng minh Câu 11. Với mỗi hoán vị p a1,a2 , ,a9 của các chữ số 1, 2, , 9, kí hiệu s p là tổng của ba số có 3 chữ số a1a2a3 , a4a5a6 , a7a8a9 . Trong các s p có hàng đơn vị bằng 0, gọi m là giá trị nhỏ nhất của nó và n là số các hoán vị p thỏa mãn s p m . Tính m n . Hướng dẫn giải Với mỗi hoán vị p a1,a2 , ,a9 của các chữ số 1, 2, , 9, kí hiệu s p là tổng của ba số có 3 chữ số a1a2a3 , a4a5a6 , a7a8a9 . Trong các s p có hàng đơn vị bằng 0, gọi m là giá trị nhỏ nhất của nó và n là số các hoán vị p thỏa mãn s p m . Tính m n . Để s p đạt giá trị nhỏ nhất thì 3 chữ số hàng trăm là 1, 2, 3, s p có chữ số tận cùng bằng 0 thì các chữ số hàng đơn vị có tổng là bội của 10. Và từ các chữ số 4, 5, 6, 7, 8, 9 không có ba số nào có tổng bằng 10 và vì 7 8 9 24 30 nên 3 chữ số hàng đơn vị phải có tổng bằng 20, ta thấy 5 6 9 4 7 9 5 7 8 20 , có ba bộ số có thể xếp vào 3 chữ số ở hàng đơn vị, tương ứng các chữ số còn lại sẽ là hàng chục. Do đó giá trị nhỏ nhất của s p là m 1 2 3 100 19 10 20 810 Như vậy có 3 trường hợp, trong mỗi trường hợp có 6 cách chọn 3 chữ số hàng trăm, 6 cách chọn 3 chữ số hàng chục và 6 cách chọn 3 chữ số hàng đơn vị. Vậy số các hoán vị p thỏa mãn yêu cầu bài toán là n 3 6 6 6 648 . Vậy m n 162 .
  7. Câu 12. Cho x1, x2 , x3,, xn là các số nguyên thỏa mãn điều kiện: 2 2 2 3 2 x1 x2 xn n 2n 1 x1 x2 xn n Chứng minh tổng S x1 x2  xn n 1 không là số chính phương. Hướng dẫn giải Câu 13. Cho trước các số nguyên dương a,b. Chứng minh rằng phương trình x2 2axy a2 4b y2 4by z2 có vô số nghiệm nguyên dương. Hướng dẫn giải Câu 14. Một số nguyên dương n được gọi là có tính chất P nếu nó thỏa mãn với mọi số nguyên dương a tùy ý, n chia hết an 1 thì n2 cũng chia hết an 1. Chứng minh rằng có vô số hợp số có tính chất P. Hướng dẫn giải Kí hiệu pk là số nguyên tố thứ k . Với mọi k 3 ta có pk 5, như vậy luôn tồn tại một số qk là ước số nguyên tố nhỏ nhất của pk 2 . n Đặt n pk qk , ta sẽ chứng minh n có tính chất P. Thật vậy, giả sử a  1(mod n) . (n) Xét hàm Ơle (n) ( pk 1)(qk 1) , a  1(mod n) . Chú ý rằng (n; (n)) 1 nên suy ra a  1(mod n) . an 1 an 1 1  n(mod n) an 1 (a 1)(an 1 an 1 1) chia hết cho n2 Hiển nhiên n là một hợp số, và dễ thấy với hai số nguyên tố pk và pm đôi một khác nhau sẽ cho ta hai hợp số pk qk và pmqm khác nhau. Vì tập các số nguyên tố là vô hạn nên cũng có vô hạn hợp số có tính chất P Câu 15. 2 © ¬2 P x 1 «P x ® 1 b. Tìm đa thức P x với hệ số thực thỏa mãn P 2014 2046 và P x P x2 1 33 32 x 0 Hướng dẫn giải a. Đặt Q x P x x thì Q 0 0. Giả sử x k là nghiệm của Q x , khi đó P k k. Suy ra, P k 2 1 k 2 1 Q k 2 1 0 mà k 2 1 k nên Q x 0 có vô số nghiệm. Do đó, Q x 0 hay P x x. b. Đặt Q x P x 32 thì Q 2014 2014 và 2 © ¬2 Q x 1 «Q x ® 1 Suy ra, Q x x hay P x x 32.
  8. Câu 16. Trên bảng có một số nguyên. Người ta ghi nhớ chữ số cuối cùng của số này, sau đó xoá đi và cộng thêm vào với số còn lại trên bảng 5 lần chữ số vừa xoá. Ban đầu có số 72014 . Hỏi có thể hay không sau một số lần thực hiện như thế thế ta thu được số 20147 ? Hướng dẫn giải Giả sử số đang có trên bảng là N 10a b 0 b 9 . Khi đó số nhận được sau một phép biến đổi là N a 5b Ta thấy 5N N 49a . Như vậy nếu N 7 thì N 7 . Do 72014 7 , còn 20147 không chia hết cho 7 nên qua các phép biến đổi đã cho từ 72014 không thể thu được 20147 . Câu 17. Cho phương trình x2 x 1 0 với là số nguyên dương. Gọi  là nghiệm dương của phương trình. Dãy số xn được xác định như sau x0 , xn 1  xn , n 0,1,2,3, Chứng minh rằng tồn tại vô hạn số tự nhiên n sao cho xn chia hết cho . Hướng dẫn giải Đầu tiên ta chứng minh  là số vô tỉ. Thật vậy, nếu  là số hữu tỉ thì  là số nguyên (do hệ số cao nhất của x2 là 1) và  là ước của 1. Do đó  1 suy ra 0 , trái giả thiết. Do đó  xn 1   xn 1  xn 1  1 xn  xn 1 xn 1 x x 1 1 x n x n x n x  n 1   n 1   n 1 xn 2 1 xn 1 1 (1). Lại có   1 0 , suy ra    xn xn xn  xn xn xn 1 xn xn xn xn 1 1 (do (1))    * Vậy xn 1  xn 1 1 (mod ) . Từ đó bằng quy nạp ta có với mọi k ¥ , n 2k 1, thì xn 1  xn (2k 1) (k 1) (mod ) (2) Chọn k + 1 = la (l Î ¥ * ), n + 1 = 2la , từ (2) ta có x2l  x0 l l  0 (mod ) . * Vậy x2l chia hết cho , l ¥ . Câu 18. Với mỗi số nguyên dương m, kí hiệu C(m) là số nguyên dương k lớn nhất sao cho luôn tồn tại một tâp S gồm m số nguyên dương để mỗi số nguyên chạy từ 1 đến k hoặc thuộc S hoặc là tổng hai phần tử thuộc S (hai phần tử này không nhất thiết phân biệt). Chứng minh: m(m 6) m(m 3) C(m) 4 2 Hướng dẫn giải Trước tiên ta tính thử một vài giá trị ban đầu của C(m) để cảm nhận bài toán. Dễ thấy: C(1)=2; C(2)=4; C(3)=8
  9. Nhận xét: Việc tính C(m) quy về việc đếm số phần tử của tập A xác định bởi: A S  (S S) ; S S x y | x, y S m(m 3) C(m) +) Chứng minh: 2 | S | (| S | 3) m(m 3) | A | | S | | S S | | S | | S | C 2 |S| 2 2 Chú ý: Để đánh giá số phần tử của tập S+S ta chia hai trường hợp x trùng y và x khác y. Rõ ràng {1;2;3;.;k} là một tập con của A nên ta được đpcm. m(m 6) C(m) +) Chứng minh: 4 Ta sẽ chỉ ra một tập B sao cho với mọi số nguyên chạy từ 1 đến m(m+6)/4 hoặc thuộc B hoặc là tổng hai số (không nhất thiết phân biệt) thuộc S(m). Khi đó C(m)>=m(m+6)/4. Xét hai trường hợp sau: TH1: m = 2n. Xét tập B(m) = {1; 2; 3;.; n; 2n+1; 3n+2;.; (n+1)n+n} gồm m phần tử và dễ thấy tập B  (B B) chứa dãy số liên tiếp từ 1 đến (n+1)n + 2n và rõ ràng (n+1)n + 2n = 2n(2n+6)/4 TH2: m = 2n+1 Khi đó ta xây dựng tập B(m)={1;2;3;.; n+1;2n+3;3n+5;.;(n+1)n+2n+1}gồm m phần tử và tập B  B B chứa dãy số liên tiếp từ 1 đến (n+1)n+3n+2 và rõ ràng (n+1)n+3n+2 > (2n+1)(2n+7)/4 Từ hai TH trên ta được đpcm. Câu 19. Cho m > 1 là một số nguyên. Chứng minh rằng với mọi số nguyên n có thể biểu diễn dưới dạng n = a + b, trong đó a là một số nguyên nguyên tố cùng nhau với m và b là một số nguyên sao cho b2  b mod m . Hướng dẫn giải - Ta xét trường hợp thứ nhất với m p , trong đó p là số nguyên tố và 1 . Giả sử n là một số nguyên. Khi đó, xảy ra hai trường hợp: a) p không chia hết n. Trong trường hợp này thì ƯCLN m,n 1 và ta có thể chọn a,b n,0 . b) p chia hết n. Trong trường hợp này p không chia hết n - 1 (bởi vì ngược lại p sẽ chia hết 1). Từ đây suy ra ƯCLN m,n 1 1 và ta có thể lấy a,b n 1,1 1 2 r - Ta đi đến trường hợp tổng quát với m p1 p2 pr với các số nguyên tố phân biệt p 1, p2, p3,., pr và các số nguyên dương 1, 2 , , r . Giả sử n là một số nguyên. Bằng cách sử dụng trường hợp ở trên, với mỗi k sao cho k 2 1 k r , thì có ak ,bk để n ak bk , ƯCLN pk ,ak 1 và bk  b mod m k -Theo định lí Trung Hoa về phần dư thì tồn tại một số nguyên b sao cho b  bk mod pk với k = 1, 2,., r 2 2 k Vì b b  bk bk  0 mod pk và p1, p2 , , pr  là các số nguyên tố phân biệt nên ta kết 2 1 2 r luận rằng b b  0 mod p1 p2 pr , tức là b2  b mod m .
  10. Câu 20. Điền 29 số nguyên dương đầu tiên vào các ô vuông con của bảng 6 x 5 bằng cách sau: Cho phép thay đổi vị trí của các số trong bảng theo quy tắc: Mỗi lần, lấy một số nằm ở ô kề với ô trống rồi chuyển số đó sang ô trống. Hỏi bằng cách thực hiện liên tiếp một số hữu hạn lần phép chuyển số nói trên đối với bảng số ban đầu ta có thể nhận được bảng số sau đó hay không? 1 2 3 4 5 29 2 3 4 5 6 7 8 9 10 6 7 8 9 10 11 12 13 14 11 12 13 14 15 16 17 18 19 15 16 17 18 19 20 21 22 23 24 20 21 22 23 24 25 26 27 28 1 25 26 27 28 29 Bảng 2 Bảng 1 Hướng dẫn giải Giả sử nhờ phép chuyển số theo qui tắc của đề bài, từ Bảng 1 ta có thể nhận được Bảng 2 (*) Ta coi ô trống của mỗi bảng là ô được điển số 0. Với mỗi bảng số nhận được trong quá trình chuyển số, ta liệt kê tất cả các số trong bảng theo thứ tự từ hàng trên xuống hàng dưới và trong mỗi hàng thì từ trái qua phải. Khi đó ứng với mỗi bảng số ta sẽ có một hoán vị của 30 số tự nhiên đầu tiên. Và do đó, điều giả sử (*) tương đương với: Từ hoán vị (1, 2, 3, 4, 5, 6, 7, 8, 9, 10, 11, 12, 0, 13, 14, 15, 16, 17, 18, 19, 20, 21, 22, 23, 24, 25, 26, 27, 28, 29) (gọi là hoán vị a) có thể nhận được hoán vị (29, 2, 3, 4,.,11. 12, 0, 13, 14, 15,.27, 28, 1) (gọi là hoán vị b) nhờ việc thực hiện liên tiếp một số hữu hạn lần phép đổi chỗ hai số trong hoán vị theo qui tắc: Mỗi lần, lấy hai số 0 của hoán vị rồi đổi vị trí của số 0 đó cho một số liền kề với số 0 đó. (1) +) Giả sử (a1, a2, a3, , a30) là một hoán vị của 30 số tự nhiên đầu tiên. Ta gọi cặp số ai ;a j là cặp số ngược của hoán vị vừa nêu nếu ai a j và i j . Dễ thấy, sau một lần thực hiện phép đổi chỗ hai số kề nhau đối với hoán vị (a 1, a2, a3, , a30) thì số cặp số ngược của hoán vị đó sẽ tăng hoặc giảm đi một đơn vị. +) Khi chuyển chỗ hai số ai và ai n ( n 1 tùy ý) trong một hoán vị, cũng tức là chuyển ai liên tiếp qua n số kề với nó và chuyển ai n liên tiếp qua n – 1 số kề với nó, nghĩa là chuyển 2n – 1 (một số lẻ lần) hai số kề nhau, do đó cặp số ngược của hoán vị đó sẽ tăng hoặc giảm một số lẻ đơn vị. (2) +) Ta có: Số cặp số ngược của của hoán vị a là 12 và số cặp số ngược của hoán vị b là 67. Từ đó, kết hợp với (2), suy ra từ hoán vị a ta chỉ có thể nhận được hoán vị b sau một số lẻ lần thực hiện phép đổi chỗ hai số nào đó. Điều này cho thấy, nếu từ Bảng 1 ta nhận được Bảng 2 thì số lần đổi chỗ hai số ở hai ô nào đó phải là số lẻ. (3) +) Tô màu tất cả các ô vuông con của bảng 6 x 5 bởi hai màu xanh, đỏ sao cho hai ô kề nhau có màu khác nhau. Sau mỗi lần đổi chỗ hai số ở hai ô kề nhau, trong đó có số 0 ở ô trống, theo cột hay theo hàng thì số 0 được chuyển từ ô có màu này sang ô có màu kia. Và vì thế do số 0 ở bảng 1 và số 0 ở bảng 2 nằm ở hai ô cùng màu nên từ bảng 1 ta chỉ nhận được bảng 2 sau một số chẵn lần đổi chỗ hai số ở hai ô kề nhau, trong đó có số 0. Điều này mâu thuẫn với (3) và mâu thuẫn đó cho thấy: Từ Bảng 1 ta không thể nhận được Bảng 2 nhờ một số hữu hạn lần đổi chỗ ở hai ô kề nhau, trong đó có số 0 ở ô trống, theo quy tắc của đề bài
  11. Câu 21. Ban đầu ta có bộ số a,b,c,d trong đó a,b,c,d là các số nguyên đôi một khác nhau. Thực hiện thuật toán sau: nếu có bộ số M x, y, z,t với x, y, z,t nguyên thì được phép thay thế x y y z z t t x bởi bộ số T M , , , . Chứng minh rằng việc thực hiện thuật toán trên 2 2 2 2 sẽ dừng lại sau hữu hạn bước. Hướng dẫn giải Giả sử ngược lại, ta nhận được bộ số với các thành phần là số nguyên. Gọi Sn max xn yn , yn zn , zn tn , tn xn , xn zn , yn tn  trong đó xn , yn , zn ,tn là bộ số nhận được sau bước thứ n . Ta có: Sn 1 Sn ,n 1. * Do Sn ¢ ,n 1 nên tồn tại m ¥ sao cho Sm 0 . Khi đó ta có: xm ym zm tm 0 . Đặt xm ym zm tm k . x y y z z t t x Ta có: m 1 m 1 m 1 m 1 m 1 m 1 m 1 m 1 k . 2 2 2 2 Suy ra: xm 1 zm 1, ym 1 tm 1 . Đặt xm 1 zm 1 u, ym 1 tm 1 v . x y z t y z t x Ta có: m 2 m 2 m 2 m 2 u; m 2 m 2 m 2 m 2 v . 2 2 2 2 x y z t Suy ra: u v m 2 m 2 m 2 m 2 . Thế thì x y z t . 2 m 1 m 1 m 1 m 1 Tiếp tục quá trình lập luận như trên dẫn đến a b c d . Vậy ta hoàn tất chứng minh. Câu 22. Có bao nhiêu cách phân tích 69 thành tích của 3 số nguyên dương, biêt các cách phân tích mà các nhân tử chỉ khác nhau về thứ tự thì chỉ được tính 1 lần? Hướng dẫn giải 푖 , 푖 ∈ 9 Xét phân tích 6 = (2 1.3 1)(2 2.3 2)(2 3.3 3) với 1 + 2 + 3 = 9 1 + 2 + 3 = 9 Với mỗi 1 ∈ , 0 ≤ 1 ≤ 9, có 10 ― 1 cách chọn số 2, để 1 + 2 ≤ 9 từ đó chọn 3 = 9 ― 1 ― 2. Vậy số cách chọn các bộ ( 1, 2, 3) là 10+9+.+1 = 55 cách số cách chọn các bộ ( 1, 2, 3) và ( 1, 2, 3) là 55.55 cách. Bây giờ, ta sẽ tính số các cách phân tích bị trùng nhau. +) TH1: 3 thừa số bằng nhau: 69 = (23.33)(23.33)(23.33) +) TH2: 2 thừa số bằng nhau: Câu 23. 69 = (2 .3 )(2 .3 )(29―2 .39―2 ) và (a; b) (3; 3). Câu 24. Khi đó a ∈ {0; 1; 2; 3; 4}; b ∈ {0; 1; 2; 3; 4 } và (a; b) (3; 3) → số cặp (a; b) là 5.5 – 1 =24, và 24 cặp này cho ta 24 cách phân tích thỏa mãn yêu cầu. Tuy nhiên, mỗi cặp sẽ cho 3 lần đếm trong quá trình đếm mà ta vừa nêu ở trên. +) TH3: nếu cả 3 thừa số khác nhau, thì mỗi phân tích bị đếm trùng 3!=6 lần.
  12. Vậy số cách phân tích là: 1 + 24 + (55 × 55 ― 24 × 3 ― 1):6 = 517 cách 1;2;3 Câu 25. Cho bộ số . 1) Chúng ta thực hiện phép biến đổi trên các bộ 3 số như sau: thay hai số trong chúng, ví dụ a và b, bởi a b và a b . Hỏi có thể nhận được bộ số sau: a1;b1;c1 thỏa mãn a1 b1 c1 10 sau khi thực hiện hữu hạn phép biến đổi từ bộ số ban đầu 1;2;3 ? 2) Nếu chúng ta thực hiện phép biến đổi trên các bộ 3 số như sau: thay hai số trong a b a b chúng, ví dụ a và b, bởi và . Hỏi có thể nhận được bộ số 28;4;2014 sau khi thực 2 2 hiện hữu hạn phép biến đổi từ bộ số ban đầu 1;2;3 Hướng dẫn giải Ta thực hiện theo cấu hình sau 1;2;3 3; 1;3 3;2; 4 3;2; 4 7;2; 1 a1;b1;c1 Dễ thấy: a1 b1 c1 10 Trong mọi cấu hình ta luôn có: Tổng bình phương các số là không đổi Lại có: 12 22 32 282 42 20142 Vậy câu trả lời là phủ định. Câu 26. Chứng minh rằng tồn tại vô hạn số nguyên dương m sao cho 2m 1 chia hết cho m. Hướng dẫn giải Ta có: 23 1  3 m Ta chỉ ra rằng với mọi số nguyên dương m, ta có: 23 1  3m Với m 1, khẳng định đúng Giả sử khẳng định đúng với m nguyên dương nào đó, tức là tồn tại k nguyên dương sao cho m 23 k.3m 1. Ta có: m 1 3 23 3m.k 1 33m.k 3 32m 1.k 2 3m 1.k 1 3m 1.t 1 với t là một số nguyên dương nào đó. Như vậy, khẳng định được chứng minh Câu 27. Tìm các số nguyên tố (không cần phân biệt) mà tích của chúng bằng mười lần tổng của chúng. Hướng dẫn giải Câu 28. Tìm tất cả các số tự nhiên n sao cho tìm được các số nguyên a, b, c thỏa mãn a + b + c = 0 và an + b n + cn là số nguyên tố Hướng dẫn giải Câu 29. Cho các số nguyên dương a, b, c, d thỏa mãn a2 + b2 + ab = c2 + d2 + cd. Chứng minh rằng: a + b + c + d là hợp số. Hướng dẫn giải
  13. Câu 30. Tồn tại hay không hai số nguyên dương phân biệt p, q sao cho q n n chia hết cho p n n với mọi số nguyên dương n? Hướng dẫn giải Giả sử tồn tại hai số p, q nguyên dương phân biệt sao cho q n n chia hết cho p n n với mọi số nguyên dương n, thế thì q n n > p n n q p . Giả sử a là một số nguyên tố lớn hơn q và n là số tự nhiên thỏa mãn n ( p 1)(a 1) 1. Khi đó n = (p+1)a –p n  p(mod a) (1) Vì p < q < a nên (p, a) =(q, a)=1. Theo định lý nhỏ Fermat, ta có pa 1  1(mod a) p( p 1)(a 1)  1(mod a) p( p 1)(a 1) 1  p(mod a). Do đó pn  p(mod a) (2) Từ (1) và (2) suy ra pn n  0(mod a) hay pn n a (4) Chứng minh tương tự, ta được qn  q(mod a) (3)và qn n a Từ (1) và (3) suy ra qn n  q p(mod a) (5) Từ (4) và (5) suy ra (q p)a . Điều này không thể sảy ra vì p q a Vậy không tồn tại hai số nguyên dương phân biệt p, q sao cho qn n chia hết cho pn n với mọi số nguyên dương n. Câu 31. Cho các số nguyên dương m,n; một bảng hình vuông kích thước n n được gọi là bảng “ m hoàn thiện” nếu tất cả các ô của nó được điền bởi các số nguyên không âm (không nhất thiết phân biệt) sao cho tổng các số trên mỗi hàng và mỗi cột đều bằng m. Hỏi có tất cả bao nhiêu cách lập bảng “2015-hoàn thiện” kích thước 3x3 sao cho số nhỏ nhất trong các số ở các ô trên đường chéo chính nằm ở vị trí tâm của bảng? (Ô ở đường chéo chính của bảng là ô ở vị trí giao của dòng có số thứ tự tính từ trên xuống và cột có số thứ tự tính từ trái sang bằng nhau; ô ở tâm bảng 3x3 là ô ở dòng thứ 2 và cột thứ 2). Hướng dẫn giải Ta giải bài toán trong trường hợp lập bảng “ m hoàn thiện” kích thước 3x3. Gọi x, y, z,t lần lượt là các số điền được ở đường chéo chính và ô ở vị trí dòng 1 cột 2, khi đó các số còn lại ở các ô được xác định duy nhất như hình bên dưới x t m x t m z x y t y x t z y t z m y t z Vì các số được điền là không âm và y là số nhỏ nhất trong các số ở đường chéo chính nên các điều kiện sau phải thỏa x, y, z,t 0; x t m; x t z; z y t m; x y t m z; y min x, y, z Các điều kiện trên có thể rút gọn lại thành 0 y min x, y, z; x t m; z y t *
  14. Khi đó 0 y 2 y t z x y t z x t m . Ta thấy rằng bộ bốn số không âm y;2 y t z; x y t z; x t sắp theo thứ tự tăng dần xác định duy nhất bộ các số x, y, z,t thỏa mãn * và tương ứng với một cách lập bảng “ m hoàn thiện”. Do vậy, số cách lập được là m 4 . 4 Áp dụng với được kết quả là 2019 . m 2015 4 Câu 32. Tìm tất cả các số tự nhiên n sao cho cả hai số 9n 16 và 16n 9 đều là số chính phương. Hướng dẫn giải n=0 thỏa mãn bài toán. Xét n>0, nếu cả hai số 9n 16 và 16n 9 đều là số chính phương thì số 2 2 2 2 An (9n 16)(16n 9) (12n) (9 16 )n 12 cũng là số chính phương. Mặt khác ta lại có (12n 12)2 (12n)2 (92 162 )n 122 (12n 15)2 2 2 Thế nên ta phải có An (12n 13) hoặc An (12n 14) , từ đó thay vào giải ra hai trường hợp ta được n 1;52 . Vậy có ba giá trị của n thỏa mãn là 0;1;52. Câu 33. Chứng minh rằng trong bảy số chính phương tuỳ ý luôn có hai số khác nhau mà hiệu của chúng chia hết cho 20. Hướng dẫn giải Dễ thấy, với mọi số chính phương khi chia cho 20 chỉ có thể dư 1; 2; 4; 8; 16. Nếu ta có 7 số chính phương thì theo nguyên lý Đirichlê, sẽ tồn tại ít nhất hai số chính phương khác nhau trong 7 số chính phương đó, có cùng số dư khi chia cho 20. Câu 34. Cho a, b, c là các số thực thỏa mãn. Chứng minh rằng đa thức có một nghiệm trong đoạn. Hướng dẫn giải Đặt. Khi đó là hàm liên tục trên R và ta có . Do vậy 1 0 9a 11b 29c f 0 f 2 a 9b 27c f 0 f 2 27 f . 3 1 Nếu một trong các số f 0 , f 2 , f bằng 0 thì hiển nhiên phương trình có một nghiệm 3 trong đoạn Nếu các số này đều khác 0 thì hai trong chúng phải khác dấu và do là hàm liên tục nên phương 1 1 trình có một nghiệm thuộc một trong các đoạn 0; , ;2 ;0;2 . Do đó nó có có một 3 3 nghiệm thuộc đoạn 0;2 Câu 35. Tìm đa thức P(x) thoả mãn điều kiện: P(1) 2013 (x y)P(x y) (x y)P(x y) 4xy(x2 y2 );x, y ¡ Hướng dẫn giải Từ giả thiết: (x y)P(x y) (x y)P(x y) 4xy(x2 y2 );x, y ¡ , thay y = x ta được P(0) = 0. Đặt P(x) = xQ(x) thì (x y)P(x y) (x y)P(x y) 4xy(x2 y2 )
  15. (x2 y2 )Q(x y) (x2 y2 )Q(x y) 4xy;x, y ¡ Q(x y) Q(x y) 4xy;x, y ¡ , x y Q(x y) Q(x y) 4xy;x, y ¡ . Thay y = x ta được Q(2x) Q(0) 4x2 Q(x) x2 Q(0);x ¡ . Ta có: Q(1) 1Q(1) P(1) 2013 Q(0) 2012 Vậy Q(x) x2 2012 P(x) x3 2012x Thử lại ta được P(x) x3 2012x thoả mãn bài toán. Câu 36. Tìm tất cả các số nguyên a sao cho tồn tại đa thức P(x) với hệ số nguyên thỏa mãn P( 3 a2 3 a ) 2 3 a2 33 a . Hướng dẫn giải Dễ thấy nếu a là lập phương của một số nguyên thì a thỏa mãn các yêu cầu của bài toán.Bây giờ ta xét trường hợp a không phải là lập phương của một số nguyên. Ta cần ba bổ đề sau Bổ đề 1.Nếu x , y , z là các số nguyên thỏa mãn x y 3 a z 3 a2 0 thì x y z 0 . Bổ đề 2. Nếu f (x) là một đa thức với hệ số nguyên thì tồn tại duy nhất bộ ba (x, y, z) các số nguyên sao cho f ( 3 a2 3 a ) x y 3 a z 3 a2 . Bổ đề 3. Nếu f (x) là một đa thức với hệ số nguyên và f ( 3 a2 3 a ) x y 3 a z 3 a2 (x, y, z ¢ ) thì y  z mod a 1 . Quay trở lại bài toán, giả sử a thỏa mãn các yêu cầu của bài toán. Ta có P( 3 a2 3 a ) 2 3 a2 33 a với P là một đa thức với hệ số nguyên. Áp dụng bổ đề 3 cho P ta có 3  2 mod a 1 , suy ra a 2 (ta đang xét a không phải là lập phương của một số nguyên.) Ngược lại, với a 2 ta có thể chọn P(x) x2 x 4 đểcó (*) . Vậy các giá trị a phải tìm là a 2 hoặc a là lập phương của một số nguyên. Câu 37. Cho p và q là các số nguyên dương khác 1. Tìm số nguyên dương m bé nhất sao cho trong mỗi m số nguyên phân biệt thuộc đoạn [ q; p], tồn tại 3 số khác nhau có tổng bằng 0. Hướng dẫn giải Ta sẽ chứng minh giá trị nhỏ nhất của m là max( p,q) , ở đây 3 nếu p , q là số chẵn và p q , c 2 trong các trường hợp còn lại. Rõ ràng giá trị nhỏ nhất của m là tồn tại, ký hiệu nó bởi m0 . Không mất tính tổng quát ta có thể giả sử p q . Đoạn [ p;q] chứa q 1 số không âm và không có ba số nào có tổng bằng 0, suy ra trong mỗi trường hợp m0 q 2 . Khi p , q là số chẵn và p q thì tập { q; q 1;L; q / 2;q / 2;L;q} gồm q 2 số và không có ba số nào có tổng bằng 0. Do đó m0 q . Bây giờ ta đi chứng minh m0 q . Xét ba trường hợp Trường hợp 1: p và q là hai số lẻ bằng nhau. Bằng quy nạp theo q ta chứng minh được Bổ đề. Nếu X  [ q;q] ‚ {0} và X không chứa ba phần tử có tổng bằng 0thì | X | q 1. Thật vậy, khẳng định hiển nhiên đúng với q 1. Giả sử nó đúng với các số lẻ bé hơn q . Xét tập X  [ q;q] ‚ {0} sao cho X không chứa ba phần tử có tổng bằng 0.
  16. Đặt Y X { q; (q 1);q 1;q}. Nếu |Y | 2 thì | X | (q 1) 2 q 1theo giả thiết quy nạp. Bây giờ ta xem là |Y | 3. Chắc chắn Y chứa hai phần tử cùng dấu, giả sử hai phần tử đó q 1 âm, do đó q Y  X . Suy ra với mỗi i 1,2,L, , nhiều nhất một phần tử trong {i,q i} 2 q 1 sẽ thuộc X , suy ra X chứa nhiều nhất phần tử dương, dấu bằng có thể xảy ra chỉ khi 2 q X . q 1 Nếu q X thì bởi tính đối xứng, X chứa nhiều nhất phần tử âm, do đó | X | q 1. 2 Nếu q  X thì q 1 X do |Y | 3. Ta có X không thể chứa cả hai i và q 1 i với q 3 q 1 i 1,2,L, . Mà X chứa q và q 1, X chứa nhiều nhất phần tử dương, và X có 2 2 q 1 q 3 q 1 thể chứa . Suy ra | X | 2 1 q 1. 2 2 2 Vậy bổ đề được chứng minh. Bây giờ gọi Z là một tập con bất kỳ của [ p;q] sao cho | Z | q 2 . Ta sẽ chứng minh rằng Z có ba phần tử có tổng bằng 0, và do định nghĩa của m0 ta có luôn m0 q 2 q . Cố định Z , và giả sử ngược lại rằng trong Z không có ba phần tử nào có tổng bằng 0. Ta có 0  Z , vì nếu trái lại trong Z sẽ chứa ba số có tổng bằng 0 dạng 0, x, x . Áp dụng bổ đề cho tập Z ta có | Z | q 1, vô lý. Trường hợp 2: p và q là hai số chẵn bằng nhau. Gọi Z là một tập con bất kỳ của [ p;q] sao cho | Z | q 3. Ta sẽ chứng minh rằng Z có ba phần tử có tổng bằng 0, và do định nghĩa của m0 ta có luôn m0 q 3 q . Cố định Z , và giả sử ngược lại rằng trong Z không có ba phần tử nào có tổng bằng 0. Ta có 0  Z , vì nếu trái lại trong Z sẽ chứa ba số có tổng bằng 0 dạng 0, x, x . Áp dụng bổ đề cho tập Z [ q 1;q 1] ta có | Z | q 2 , vô lý. Trường hợp 3: p q . Gọi Z là một tập con bất kỳ của [ p;q] sao cho | Z | q 2 . Ta sẽ chứng minh rằng Z có ba phần tử có tổng bằng 0, và do định nghĩa của m0 ta có luôn m0 q 2 q . Cố định Z , và giả sử ngược lại rằng trong Z không có ba phần tử nào có tổng bằng 0. Ta có 0  Z , vì nếu trái lại trong Z sẽ chứa ba số có tổng bằng 0 dạng 0, x, x . Nếu q chẵn thì X ‚ {q}  [ q 1;q 1] , theo trường hợp 1 ta có | X ‚ {q}| q , suy ra | X | q 1, vô lý. Nếu q lẻ thì X  [ q;q], theo trường hợp 1 ta có | X ‚ {q}| q , suy ra | X | q 1, vô lý. Bài toán được giải hoàn toàn. Câu 38. Cho tập hợp X 1;2;3; ;2016. Tìm số k nguyên dương nhỏ nhất sao cho với mọi tập con gồm k phần tử của tập hợp X đều chứa ít nhất 5 số nguyên liên tiếp. Hướng dẫn giải Xét tập hợp A X / 5k,1 k 403 A 2016 403 1613 Với k không lớn hơn 1613, thì chọn bất kỳ tập hợp B là tập con gồm k phần tử của A, cũng là tập con của X và B không thể chứa 5 số nguyên liên tiếp. (1,5 điểm)
  17. Nếu k = 1614. Xét C là một tập con của X gồm 1614 phần tử A 5i 4;5i 3;5i 2;5i 1;5i ,i 1;403, A 2016 i  404  (1.5 điểm) Nếu mỗi tập hợp trên chứa tối đa 4 phần tử thuộc C thì số phần tử của C không quá 4x403+1= 1613 ( vô lý). Vậy trong các tập hợp gồm 5 phần tử trên phải có 1 tập là con của C nên C chứa 5 số nguyên lien tiếp. Vậy số k nhỏ nhất cần tìm bằng 1614. * Câu 39. Cho n ¥ , a0 < a1 < a2 < < an là các số nguyên và P(x) là đa thức bậc n có hệ số cao nhất n! bằng 1. Chứng minh rằng tồn tại i {0, 1, , n} sao cho P a . i 2n Hướng dẫn giải Khai triển đa thức P(x) theo công thức nội suy Lagrange có: n n x a j P x  P ai  . i 0 j 0 ai a j j i Xét hệ số của xn có: n n n 1 P a P ai 1 P a 1 i  i n  n  n i 0 a a i 0 a a i 0  i j  i j  ai a j j 0, j i j 0, j i j 0, j i Đặt M max P ai , vì a0 < a1 < a2 < < an là các số nguyên nên có: 0 i n n P a n M n M 1 i  n  n  i 0 i 0 i 0 i! n i !  ai a j  ai a j j 0, j i j 0, j i n 1 1 n n! 2n 2n Lại có   . Từ đó suy ra: 1 M. i 0 i! n i ! n! i 0 i! n i ! n! n! n! n! M vì M max P ai 0 i n để P(ai ) . 2n 0 i n 2n Câu 40. Tìm tất cả các đa thức với hệ số thực thỏa mãn: P(x).P(x 1) P(x2 2) , x R Hướng dẫn giải Nếu degP = 0 thì P(x)  c , c là hằng số Từ (1) c.c c c 0 c 1 Suy ra trường hợp này có hai đa thức: P(x)  0, P(x)  1 thỏa đề ra . Nếu degP = m với m lẻ thì đa thức P(x) luôn có 1 nghiệm x0 R 2 2 Từ (1) suy ra: P(x0 2) =P(x0 ).P(x0 1) 0 suy ra x0 2 cũng là nghiệm của P(x) u x Xét dãy số: (u ) : 1 0 n u u2 2 , n 2 n n 1 Dễ dàng ta thấy (un ) là dãy tăng và bằng quy nạp theo n ta có: P(un ) 0 , n Do đó đa thức P(x) có vô số nghiệm: điều này vô lý.
  18. Vì vậy degP(x) là chẵn . Xét degP(x) = 2n, n N * 2n 2n 1 Ta viết lại P(x) a2n x a2n 1x a1x a0 , a2n 0 Từ quan hệ (1) của bài toán, ta đồng nhất hệ số của x4n ở cả hai vế phương trình hàm, ta được: 2 a2n a2n a2n 1 Ta đặt P(x) (x2 x 2)n G(x) với degG(x) 2n và G(x) 0 Khi đó: P(x).P(x 1) P(x2 2) , x R 2 n 2 n 2 2 2 2 n n G(x) (x x 2) . G(x 1) (x x 2) G(x 2) (x 2) (x 2) 2 G(x).G(x 1) G(x).(x2 x 2)n G(x 1).(x2 x 2)n G(x2 2) , (2) Vì: (x2 x 2).(x2 x 2) (x2 2)2 x2 (x2 2)2 (x2 2) 2 Mà degG(x) k 2n suy ra: VT(2) có bậc là: 2n k , VP(2) có bậc là 2k Nhưng: 2n k 2k Do đó phải có G(x)  0 , ta tìm được: P(x) (x2 x 2)n ,x R . Vậy các đa thức cần tìm là: P(x)  0, P(x)  1, P(x) (x2 x 2)n ,x R Câu 41. Tìm tất cả các đa thức hệ số thực P(x) không đồng nhất không thỏa mãn: P(2014) = 2046, P(x) P(x2 1) 33 32,x 0 Hướng dẫn giải Giả sử P(x) thỏa mãn đầu bài. Khi đó ta có P(x2 1) [P(x) 32]2 33,x 0 2 2 2 Suy ra P(2014 1) (2046 32) 33 2014 33 Đặt x0=2014, ta có x0 32 2046, P(x0 ) x0 32 do P(2014) = 2046. 2 2 Xét dãy {xn} như sau: x0=2014, x1 x0 1, xn 1 xn 1, n=1,2,3 Khi đó P(x0 ) x0 32 2 2 2 2 P(x1) P(x0 1) [P(x0 ) 32] 33 x0 33 x0 1 32 x1 32 2 2 2 2 P(x2 ) P(x1 1) [P(x1) 32] 33 x1 33 x1 1 32 x2 32 Bằng quy nạp dễ dàng chứng minh được P(xn ) xn 32,n 0,1,2 (*) Xét đa thức hệ số thực Q(x) P(x) x 32 Từ (*) ta có Q(x) nhận xn làm nghiệm với mọi n=0,1,2 Mặt khác do dãy {xn}n 0 tăng nghiêm ngặt nên Q(x)  0 suy ra P(x) = x+32 Thử lại ta có P(x) thỏa mãn đầu bài. Vậy: Có duy nhất đa thức P(x) = x + 32 Câu 42. Cho đa thức P x 4x3 54x2 243x m , với m ¢ . Chứng minh rằng tồn tại n ¢ sao cho P n 821 với mọi m. Hướng dẫn giải
  19. Nhận xét 821 là một số nguyên tố có dạng 3k 2 . Để chứng minh bài toán ta chứng minh A P 1 , P 2 ,, P 821  là một hệ đầy đủ mod821 với mọi m. Nghĩa là P ni  P n j mod821 thì ni  n j mod821 Vì 2,821 1 nên P ni  P n j mod821 2P ni  2P n j mod821 . 3 2 3 2 2 4ni 54ni 243ni m  2 4n j 54n j 243n j m mod821 3 3 2ni 9  2n j 9 mod821 (1), với mọi m. Ta chứng minh bổ đê sau; nếu x3  y3 mod p thì x  y mod p với p 3k 2 là một số nguyên tố. Thật vậy. Nếu x  0 mod p x3  0  y3 mod p y  0 mod p x  y mod p Nếu x  p hay x, p 1 và y, p 1 , theo Fermat ta có x3k 1  1  y3k 1 mod p Từ x3  y3 x3k xy  y3k xy x3k 1 y  y3k 1x y  x mod p Vậy từ (1) 2ni 9  2n j 9 mod821 ni  n j mod821 , và vì 1 ni ,n j 821 nên ni n j . Vậy A P 1 , P 2 ,, P 821  là một hệ đầy đủ mod821 với mọi m. Suy ra với mọi m, tồn tại ni sao cho P ni A thỏa mãn P ni 821. 3 2 Câu 43. Cho đa thức P(x) x 2009x 2015x 2012 . * Đặt P1(x) P(x); Pn 1(x) P(Pn (x)), n N . Chứng minh rằng tồn tại vô hạn số nguyên dương n thỏa mãn Pn (x) x chia hết cho 2003 với mọi số nguyên x. Hướng dẫn giải Bổ đề: P(x)  P( y) (mod 2003) x  y (mod 2003) x,y Z Chứng minh: Ta chỉ cần chứng minh P(x)  P( y) (mod 2003) x  y (mod 2003) Thật vậy P(x) (x 2)3 2003(x2 x 1) 1 Do đó P(x)  P( y) (mod 2003) (x 2)3  ( y 2)3 (mod 2003) x,y Z Nếu (x 2)  0 (mod 2003) ( y 2)  0(mod 2003) . Do đó x  y (mod 2003) Nếu (x 2) 0 (mod 2003) ( y 2) 0(mod 2003) . Áp dụng định lý Fermat với 2003 là số nguyên tố ta có: (x 2)2002  ( y 2)2002  1 (mod 2003) Mặt khác (x 2)3  ( y 2)3 (mod 2003) (x 2)2001  ( y 2)2001 (mod 2003) (x 2)2002  ( y 2)2001(x+2) (mod 2003) ( y 2)2002  ( y 2)2001(x+2) (mod 2003) (x 2)  ( y 2) (mod 2003) x  y (mod 2003) (đpcm). Trở lại bài toán: Đặt A={0;1;2;.2002}. Với mỗi x A , xét dãy P1(x), P2(x).,P2004(x). Theo nguyên lý Dirichle tồn tại các số m,k thỏa mãn 1 m k 2004 và Pm (x)  Pk (x) (mod2003) suy ra Pm (x)  Pm (Pk m (x)) (mod2003) Áp dụng bổ đề ta có Pk m (x)  x (mod2003) . Vì vậy với mỗi x A luôn tồn tại n N * thỏa mãn: P (x)  x (mod 2003) . x nx
  20. Lấy một bội số chung n > 1 của n 0, n1,.n2012. Ta sẽ chứng minh n là một giá trị thỏa mãn yêu cầu cầu của đề bài. Thậy vậy với x A ta dễ thấy P (x) P (x)  P (x)  x (mod 2003) n knx nx Với mỗi số nguyên x A luôn tồn tại y A, x  y (mod2003) suy ra Pn (x)  Pn ( y)  y  x (mod2003) Vì vậy Pn (x) x chia hết cho 2003 với mọi số nguyên x. Vì tồn tại vô hạn bội chung n > 1 của n0, n1,.n2012 nên có vô hạn n thỏa mãn bài toán (đpcm). Câu 44. Tìm các chữ số a,b,c,d sao cho abcd2016 chia hết cho 2017 Hướng dẫn giải Đặt abcd A,1000 A 9999 . Ta có abcd20162017 (10000A 2016)2017 (4.2017A 1932A 2017 1)2017 (1932A 1)2017 1139(1932A 1)2017 (*) Vì (1139, 2017) = 1 nên (*) (1091.2017A A 1139)2017 (A 1139)2017 . Vì 1000 A 9999 nên 139 A 1139 8806 . Do đó A 1139 0 A 1139 A 1139 2017 A 3156 (A 1139)2017 A 1139 4034 A 5173 . A 1139 6051 A 7190 A 1139 8068 A 9207 Từ đây suy ra các chữ số a,b,c,d .